Re: [obm-l] integral euleriana, funcao gama

2006-01-31 Por tôpico Marcelo Salhab Brogliato

Opa..
Então, eu fiz o processo inverso:

Gamma(x) = int(0 to +inf, t^(x-1) e^(-t) dt )

Gamma((m+1)/n) = int(0 to +inf, t^((m-n+1)/n) e^(-t) dt )

t = (x-a)^n
dt = n(x-a)^(n-1)dx

t-0   x-a
t-+inf  x-+inf

Gamma((m+1)/n) = int(a to +inf, (x-a)^(m-n+1) n (x-a)^(n-1) e^(-(x-a)^n) 
dx )

Gamma((m+1)/n) = int(a to +inf, n (x-a)^m e^(-(x-a)^n) dx )

Logo:

(1/n) . Gamma[(m+1)/n] = int(a to +inf, (x-a)^m . e^[-(x-a)^n] . dx )

Agora é necessário mostrar que essa integral, com limites de a até 0, vale 
0.


O ponto é: se a for inteiro, entao, a integral de a até 0 vale: -a! 
[fatorial]


Tem certeza dessa questao?
a, m e n são reais?

Se sim, peço que me ajude a achar meu erro.. qquer idéia é bem vinda.

Abraços,
Salhab

- Original Message - 
From: Luís [EMAIL PROTECTED]

To: obm-l@mat.puc-rio.br
Sent: Monday, January 30, 2006 12:49 PM
Subject: [obm-l] integral euleriana, funcao gama


provar que:
integ(zero a infinito)[(x - a)^m . e^[-(x - a)^n] . dx = (1/n) . Gama[(m + 
1)/n]
eu tentei algumas substituicoes mas nunca consegui acertar os limites,abaixo 
um exemplo que nao da certo:

x - a = b x - 0, b - -adx = dbx - inf, b - inf
fica:integ( -a a zero)[b^m . e^[-(b)^n] . db + integ(zero a infinito)[(b)^m. 
e^[-(b)^n] . db
sendo que a segunda integral, fazendo nova substituicao parecida, 
ehjustamente o resultado, entao a primeira teria que valer zero, mascomo 
provar?

alguma ideia melhor?
=
Instrues para entrar na lista, sair da lista e usar a lista em
http://www.mat.puc-rio.br/~nicolau/olimp/obm-l.html
= 


=
Instru��es para entrar na lista, sair da lista e usar a lista em
http://www.mat.puc-rio.br/~nicolau/olimp/obm-l.html
=


[obm-l] Algebra

2006-01-31 Por tôpico gustavo




Quem puder ajudar , obrigado !!

1) Se x+y+Z = 0 e x^2 + y^2 +z^2 = 1 , Calcule A = 
x^4 + y^4 +z^4 . (m^p é m elevado a p)

2)Qual as soluçôes inteiras e positivas da equação 
a^3 - b^3 = 602



[obm-l] ESTRATÉGIA VENCEDORA!

2006-01-31 Por tôpico Jorge Luis Rodrigues e Silva Luis

Ok! Benedito e demais colegas!

São dados um tabuleiro de xadrez 8*8 e palitinhos do tamanho dos lados das 
casas. Dois jogadores jogam alternadamente e, em cada jogada, um dos 
jogadores coloca um palitinho sobre um lado de uma casa do tabuleiro, sendo 
proibido superpor palitinhos. Vence o jogador que conseguir completar 
primeiro um quadrado 1*1 de palitinhos. Supondo que nenhum jogador cometa 
erros, qual dos dois jogadores tem a estratégia vencedora, ou seja, consegue 
vencer independentemente de como jogue seu adversário?


Dois meninos jogam o seguinte jogo. O primeiro escolhe dois números inteiros 
diferentes de zero e o segundo monta uma equação do segundo grau usando como 
coeficientes os dois números escolhidos pelo primeito jogador e 1998, na 
ordem que quiser (ou seja, se o primeiro jogador escolhe a e b o segundo 
jogador pode montar a equação 1998x^2+ax+b=0 ou bx^2+1998x+a=0, etc.) O 
primeiro jogador é considerado vencedor se a equação tiver duas raízes 
racionais diferentes. Mostre que o primeiro jogador pode ganhar sempre.


Abraços!

_
Inscreva-se no programa beta do novo Windows Live Mail e seja um dos 
primeiros a testar as novidades. Saiba mais: 
http://www.ideas.live.com/programpage.aspx?versionId=5d21c51a-b161-4314-9b0e-4911fb2b2e6d


=
Instruções para entrar na lista, sair da lista e usar a lista em
http://www.mat.puc-rio.br/~nicolau/olimp/obm-l.html
=


[obm-l] PROBLEMAS FALACIOSOS!

2006-01-31 Por tôpico Jorge Luis Rodrigues e Silva Luis
Creuza tinha em mãos dois vidros do mesmo tamanho. Um pela metade de água e 
o outro de óleo. Creuza despejou a metade do óleo no vidro que continha água 
e, desta mistura transferiu quantidades equivalentes ao vidro com óleo, de 
maneira que todos ficaram com a mesma quantidade de líquido. Desta forma o 
vidro que continha inicialmente apenas óleo, ficou com mais óleo ou de água 
com mais óleo?


NOTA: Inexplicavelmente, esta é a pegadinha mais ardilosa que conheço, pois 
até esta data, ninguém respondeu corretamente. Pasmem! O vidro que continha 
óleo ficou com mais óleo.



Um galinheiro que havia sòmente patas diferentes, foi invadido por uma 
raposa faminta que deixou como vestígio algumas patas, de forma que se 
reunirmos em pares, combinam ou um só par de patas ou todos os pares. 
Afinal! Quantas aves foram mortas?


NOTA: Este é outro campeão de erros, devido constar no enunciado dois tipos 
de patas, pata=ave e pata=pé. Cabe ao aluno identificar adequadamente, já 
que o enunciado dispõe de ferramentas...



Cada um deve receber uma primeira parte que, a cada vez, tenha um real a 
mais do que irá receber, também na primeira parte, o precedente. Depois, 
deve receber mais uma segunda parte, que terá de ser igual à sexta parte da 
sobra verificada na ocasião de receber esta segunda parte. Terminada a 
distribuição, não sobrará nenhum real. Qual o total?


NOTA: Por motivo ignorado, este se encontra há décadas em aberto na 
lista...Estranho, não!


Divirtam-se!

_
Ganhe tempo encontrando o arquivo ou e-mail que você precisa com Windows 
Desktop Search. Instale agora em  http://desktop.msn.com.br


=
Instruções para entrar na lista, sair da lista e usar a lista em
http://www.mat.puc-rio.br/~nicolau/olimp/obm-l.html
=


[obm-l] funçao geradora ordinaria!!!

2006-01-31 Por tôpico diego andres
gostaria que alguem achasse a funcao geradora da sequencia(1,1,1,0,0,0,1,1,1,0,0,0...). grato Diego Andrés 
		 
Yahoo! doce lar. Faça do Yahoo! sua homepage.

Re: [obm-l] Algebra

2006-01-31 Por tôpico profmarcio
Consegui alguma coisa na 2). Mas, pelo trabalho que dá, eu desconfio que alguém aparecerá com uma alternativa mais simples.Enquanto isso, dá uma olhada no meu servio braal aí embaixo.a^3 - b^3 = (a - b)(a^2 + ab + b^2)Divisores de 602: 1, 2, 7, 14, 43, 86, 301 e 602 Seja a - b = k, k um divisor de 602.(k + b)^2 + (k + b)b + b^2 = 602/k == k^2 + 2kb + b^2 + kb + b^2 + b^2 = 602/k == 3b^2 + 3kb + p^2 - 602/k = 0Discriminante = D = 12(602/k) - 3k^2Testando para quais dentre os possíveis valores de k obtemos um D quadrado perfeito, encontramos k = 2, e daí, b = 9 e a = 11.Essa é a única soluo inteira e positiva.Abraos,Márcio.On Mar Ene 31  9:29 , 'gustavo'  sent:





 
Quem puder ajudar , obrigado !!
 
1) Se x+y+Z = 0 e x^2 + y^2 +z^2 = 1 , Calcule A = 
x^4 + y^4 +z^4 .(m^p é m elevado a p)
 
2)Qual as solues inteiras e positivas da equao 
a^3 - b^3 = 602
 Instruções para entrar na lista, sair da lista e usar a lista em
http://www.mat.puc-rio.br/~nicolau/olimp/obm-l.html


Re: [obm-l] integral euleriana, funcao gama

2006-01-31 Por tôpico Luís
 Logo: (1/n) . Gamma[(m+1)/n] = int(a to +inf, (x-a)^m . e^[-(x-a)^n] . dx 
 ) Agora é necessário mostrar que essa integral, com limites de a até 0, 
 vale 0.
Mas aih estaria pressupondo que a eh menor que zero com base em q?
 O ponto é: se a for inteiro, entao, a integral de a até 0 vale: -a! 
 [fatorial]
se a for inteiro entao gama(a) = a!do jeito que a integral estah nao vejo 
motivo pra ser a mesma
 Tem certeza dessa questao?
nao vi ninguem que fez
 a, m e n são reais?
sao
=
Instru��es para entrar na lista, sair da lista e usar a lista em
http://www.mat.puc-rio.br/~nicolau/olimp/obm-l.html
=


Re: [obm-l] integral euleriana, funcao gama

2006-01-31 Por tôpico Luís
correcao:
se a for inteiro entao gama(a) = (a - 1)!
e NAO a!
=
Instru��es para entrar na lista, sair da lista e usar a lista em
http://www.mat.puc-rio.br/~nicolau/olimp/obm-l.html
=


Re: [obm-l] Algebra

2006-01-31 Por tôpico Eduardo Wilner
 Para a 1) pode-se fazer  1 = (x^2+y^2+z^2)^2 =A+2B  (I)  onde B=x^2 y^2 +x^2 z^2 +y^2 z^2,e 0 = (x+y+z)^4 = (1+2(xy + xz + yz))^2 (II). A (II) pode ser usada duas vezes = 0 = 1 + 4B + 4C onde C=xy+xz+yz  e  0 = (1+2C)^2  = C = - 1/2 . Daí chega-se em A = 1/2. [EMAIL PROTECTED] escreveu:  Consegui alguma coisa na 2). Mas, pelo trabalho que dá, eu desconfio que alguém aparecerá com uma alternativa mais simples.Enquanto isso, dá um!
a olhada
 no meu "serviço braçal" aí embaixo.a^3 - b^3 = (a - b)(a^2 + ab + b^2)Divisores de 602: 1, 2, 7, 14, 43, 86, 301 e 602 Seja a - b = k, k um divisor de 602.(k + b)^2 + (k + b)b + b^2 = 602/k == k^2 + 2kb + b^2 + kb + b^2 + b^2 = 602/k == 3b^2 + 3kb + p^2 - 602/k = 0Discriminante = D = 12(602/k) - 3k^2Testando para quais dentre os possíveis valores de k obtemos um D quadrado perfeito, encontramos k = 2, e daí, b = 9 e a = 11.Essa é a única solução inteira e positiva.Abraços,Márcio.On Mar Ene 31  9:29 , 'gustavo'  sent:   Quem puder ajudar , obrigado !! 1) Se x+y+Z =!
 0 e x^2
 + y^2 +z^2 = 1 , Calcule A =   x^4 + y^4 +z^4 .(m^p é m elevado a p) 2)Qual as soluçôes inteiras e positivas da equação   a^3 - b^3 = 602   Instruções  para entrar na lista, sair da lista e usar a lista em  http://www.mat.puc-rio.br/~nicolau/olimp/obm-l.html  
		 
Yahoo! doce lar. Faça do Yahoo! sua homepage.

[obm-l] Re: [obm-l] funçao geradora ordinaria!!!

2006-01-31 Por tôpico Marcelo Salhab Brogliato



Olá,
então, fiz o seguinte:
f(x) = x - [x] , onde [x] é o maior inteiro menor 
que x

Fiz o seguinte, fiz o grafico ir até 2, ao invés de 
1..
f(x) = 2(x - [x])

Então, estiquei para que ao inves do periodo ser 1, 
ser 3.
Então:

f(x) = 2(x/3 - [x/3])

Assim, g(x) = [f(x)] = [2(x/3 - [x/3])], x pertence 
aos naturais
nos da a seguencia:
g(0) = 0
g(1) = 0
g(2) = 0
g(3) = 1
g(4) = 1
g(5) = 1
g(6) = 0
e assim segue..
agora transladamos o grafico para 
tras..
logo:

f(x) = [ 2 ( (x+3/2)/3 - [ (x+3/2)/3 ] ) 
]
nos da a sequencia desejada.

Abraços,
Salhab


  - Original Message - 
  From: 
  diego andres 
  To: obm-l@mat.puc-rio.br 
  Sent: Tuesday, January 31, 2006 10:50 
  AM
  Subject: [obm-l] funçao geradora 
  ordinaria!!!
  
  gostaria que alguem achasse a funcao geradora da 
  sequencia(1,1,1,0,0,0,1,1,1,0,0,0...).grato Diego Andrés
  
  
  Yahoo! doce lar. Faça 
  do Yahoo! sua homepage.


Re: [obm-l] Usando integral

2006-01-31 Por tôpico saulo nilson
V =int(pi f(x)^2)dx(x^2 + y^2)^2 = 18xy
x =rcosa
y=rsena
a equaçao se reduz a:
r^2 = 9sen2a
daqui tiramos que
rdr=9cos2ada
dx = dr*coa -r*sea*da
dx =da* (9cos2a*cosa/r - rsena)
V =int(pi f(x)^2)dx= int pi* r^2sen^2a*(9cos2a*cosa/r - rsena)da a=0 a 2pi




On 1/29/06, Alexandre Bastos [EMAIL PROTECTED] wrote:


1.) Ache o volume do sólido gerado pela rotação, em torno do eixo x, da região limitada pela curvacuja equação é: (x^2 + y^2)^2 = 18xy
__Faça ligações para outros computadores com o novo Yahoo! Messenger 
http://br.beta.messenger.yahoo.com/ 


Re: [obm-l] Re:

2006-01-31 Por tôpico saulo nilson
Ola, vc pode participar de uma comunidade no orkut que se chama projeto IME, ITA, AFA, la tem muita gente boa, o pessoal resolve provas la, disponibiliza material para estudo, e tira duvidas como a sua. 
Abraço, saulo.
On 1/29/06, Miguel Almeida [EMAIL PROTECTED] wrote:

http://www.vestseller.com.br/
On 1/28/06, ivanzovisk [EMAIL PROTECTED]
 wrote: 


Olá, este ano vou tentar entrar no ITA por isso gostaria que alguém desta lista min ajudasse, indicando os melhores livros para Portugues, Ingles, Fisica, Quimica e Matematica. Muito obrigado.



Re: [obm-l] Re: [obm-l] funçao geradora ordinaria!!!

2006-01-31 Por tôpico Henrique Rennó
Olá Marcelo!!!

Acredito que a fórmula encontrada não está correta. Caso eu esteja
errado me corrija.

f(x) = [ 2 ( (x+3/2)/3 - [ (x+3/2)/3 ] ) ]

f(0):
0+3/2 = 3/2
3/2/3 = 1/2
[x] é o maior inteiro menor que x
[1/2] = 0
f(0) = [2(1/2 - 0)] = 0 -- valor incorreto

f(1):
1+3/2 = 5/2
5/2/3 = 5/6
[x] é o maior inteiro menor que x
[5/6] = 0
f(0) = [2(5/6 - 0)] = 1 -- valor correto

f(2):
2+3/2 = 7/2
7/2/3 = 7/6
[x] é o maior inteiro menor que x
[7/6] = 1
f(0) = [2(7/6 - 1)] = 0 -- valor incorreto

Acho que a fórmula abaixo pode ser usada:

f(x) = [(x+3)/3] mod 2,
onde x pertence a N e [x] é o maior inteiro menor ou igual a x

f(0) = [(0+3)/3] mod 2 = [3/3] mod 2 = 1 mod 2 = 1
f(1) = [(1+3)/3] mod 2 = [4/3] mod 2 = 1 mod 2 = 1
f(2) = [(2+3)/3] mod 2 = [5/3] mod 2 = 1 mod 2 = 1
f(3) = [(3+3)/3] mod 2 = [6/3] mod 2 = 2 mod 2 = 0
f(4) = [(4+3)/3] mod 2 = [7/3] mod 2 = 2 mod 2 = 0
f(5) = [(5+3)/3] mod 2 = [8/3] mod 2 = 2 mod 2 = 0
f(6) = [(6+3)/3] mod 2 = [9/3] mod 2 = 3 mod 2 = 1
f(7) = [(7+3)/3] mod 2 = [10/3] mod 2 = 3 mod 2 = 1
f(8) = [(8+3)/3] mod 2 = [11/3] mod 2 = 3 mod 2 = 1
f(9) = [(9+3)/3] mod 2 = [12/3] mod 2 = 4 mod 2 = 0
f(10) = [(10+3)/3] mod 2 = [13/3] mod 2 = 4 mod 2 = 0
f(11) = [(11+3)/3] mod 2 = [14/3] mod 2 = 4 mod 2 = 0
f(12) = [(12+3)/3] mod 2 = [15/3] mod 2 = 5 mod 2 = 1
.
.
.

Abraços,

On 1/31/06, Marcelo Salhab Brogliato [EMAIL PROTECTED] wrote:
 Olá,
 então, fiz o seguinte:
 f(x) = x - [x] , onde [x] é o maior inteiro menor que x

 Fiz o seguinte, fiz o grafico ir até 2, ao invés de 1..
 f(x) = 2(x - [x])

 Então, estiquei para que ao inves do periodo ser 1, ser 3.
 Então:

 f(x) = 2(x/3 - [x/3])

 Assim, g(x) = [f(x)] = [2(x/3 - [x/3])], x pertence aos naturais
 nos da a seguencia:
 g(0) = 0
 g(1) = 0
 g(2) = 0
 g(3) = 1
 g(4) = 1
 g(5) = 1
 g(6) = 0
 e assim segue..
 agora transladamos o grafico para tras..
 logo:

 f(x) = [ 2 ( (x+3/2)/3 - [ (x+3/2)/3 ] ) ]
 nos da a sequencia desejada.

 Abraços,
 Salhab

 - Original Message -
 From: diego andres
 To: obm-l@mat.puc-rio.br
 Sent: Tuesday, January 31, 2006 10:50 AM
 Subject: [obm-l] funçao geradora ordinaria!!!

  gostaria que alguem achasse a funcao geradora da
 sequencia(1,1,1,0,0,0,1,1,1,0,0,0...).
 grato Diego Andrés


 
 Yahoo! doce lar. Faça do Yahoo! sua homepage.




--
Henrique

=
Instruções para entrar na lista, sair da lista e usar a lista em
http://www.mat.puc-rio.br/~nicolau/olimp/obm-l.html
=


Re: [obm-l] PROBLEMAS FALACIOSOS!

2006-01-31 Por tôpico Chicao Valadares
 Creuza tinha em mãos dois vidros do mesmo tamanho.
 Um pela metade de água e 
 o outro de óleo. Creuza despejou a metade do óleo no
 vidro que continha água 
 e, desta mistura transferiu quantidades equivalentes
 ao vidro com óleo, de 
 maneira que todos ficaram com a mesma quantidade de
 líquido. Desta forma o 
 vidro que continha inicialmente apenas óleo, ficou
 com mais óleo ou de água 
 com mais óleo?

Se cada um começa com 1 entao no final temos 0,5 +
0,33*0,5 =0,66 de oleo  onde somente havia oleo no
inicio e 0,66*0,5= 0,33 de oleo onde somente havia
agua no inicio nao entendi a pegadinha aqui :p 

 


O Binômio de Newton é tão belo como a Vênus de Milo.
O que há é pouca gente para dar por isso... 
Fernando Pessoa - Poesias de Alvaro Campos

_
As informações existentes nessa mensagem e no(s) arquivo(s) anexado(s) 
são
para uso restrito, sendo seu sigilo protegido por lei. Caso não seja
destinatário, saiba que leitura, divulgação ou cópia são proibidas. 
Favor
apagar as informações e notificar o remetente. O uso impróprio será 
tratado
conforme as normas da empresa e a legislação em vigor. Agradecemos sua
colaboração.


The information mentioned in this message and in the archives attached 
are
of restricted use, and its privacy is protected by law. If you are not 
the
addressee, be aware that reading, disclosure or copy are forbidden. 
Please
delete this information and notify the sender. Inappropriate use will 
be
tracted according to company's rules and valid laws. Thank you for your
cooperation.








___ 
Yahoo! doce lar. Faça do Yahoo! sua homepage. 
http://br.yahoo.com/homepageset.html 

=
Instruções para entrar na lista, sair da lista e usar a lista em
http://www.mat.puc-rio.br/~nicolau/olimp/obm-l.html
=


[obm-l] Mais um Problema de Jorge ressuscitado

2006-01-31 Por tôpico Chicao Valadares
Nao lembro mais em que email ele postou esse problema:

 Mostre que a diferença entre um número racional,
suposto
distinto de zero e um, e seu inverso, nunca é um
número inteiro.

Mas ele o postou e ninguem da lista resolveu.Aqui esta
a soluçao de um colega meu de faculdade:

Seja x=a/b (com mdc(a,b)=1) o número racional em
questão e suponha que x é diferente de 0, 1 e -1.
Temos

x-1/x=a/b-b/a=(a^2-b^2)/(ab)=(a+b).(a-b)/(ab). (*)

Suponha que d é um divisor comum de a e de a+b.
Então d divide (a+b)-a=b. Como mdc(a,b)=1, temos,
necessariamente, d=1.
Analogamente (gosto dessa palavra):

mdc(a,a-b)=mdc(b,a+b)=mdc(b,a-b)=1.

Sendo assim, em (*) não existe fator comum entre
numerador e denominador. Para que x-1/x seja inteiro
restam as opções

a+b=0, a-b=0, ab=1.


1) Se a+b=0, teremos a=-b e x=a/b=-1, o que é nao pode
por hupótese.

2) Se a-b=0, teremos a=b e x=a/b=1, o que também não
pode.

3) Finalmente, se ab=1, teremos a=b=1 ou a=b=-1 e
ocorre x=a/b=1; nao pode de novo!

Sendo assim, não existe tal racional.

O Binômio de Newton é tão belo como a Vênus de Milo.
O que há é pouca gente para dar por isso... 
Fernando Pessoa - Poesias de Alvaro Campos

_
As informações existentes nessa mensagem e no(s) arquivo(s) anexado(s) 
são
para uso restrito, sendo seu sigilo protegido por lei. Caso não seja
destinatário, saiba que leitura, divulgação ou cópia são proibidas. 
Favor
apagar as informações e notificar o remetente. O uso impróprio será 
tratado
conforme as normas da empresa e a legislação em vigor. Agradecemos sua
colaboração.


The information mentioned in this message and in the archives attached 
are
of restricted use, and its privacy is protected by law. If you are not 
the
addressee, be aware that reading, disclosure or copy are forbidden. 
Please
delete this information and notify the sender. Inappropriate use will 
be
tracted according to company's rules and valid laws. Thank you for your
cooperation.







___ 
Yahoo! doce lar. Faça do Yahoo! sua homepage. 
http://br.yahoo.com/homepageset.html 

=
Instruções para entrar na lista, sair da lista e usar a lista em
http://www.mat.puc-rio.br/~nicolau/olimp/obm-l.html
=


Re: [obm-l] Mais um Problema de Jorge ressuscitado

2006-01-31 Por tôpico Henrique Rennó
Olá Chicão!!!

Não entendi uma igualdade no decorrer da explicação:

 Então d divide (a+b)-a=b. Como mdc(a,b)=1, temos,

(a+b).a=b -- Por que essa igualdade foi escolhida???

Suponha a=7 e b=2, ou seja, o racional é 7/2.

(7+2).7=2 -- 9.7=2 -- 63=2 -- ???

Agradeço a atenção,

Abraços

On 1/31/06, Chicao Valadares [EMAIL PROTECTED] wrote:
 Nao lembro mais em que email ele postou esse problema:

  Mostre que a diferença entre um número racional,
 suposto
 distinto de zero e um, e seu inverso, nunca é um
 número inteiro.

 Mas ele o postou e ninguem da lista resolveu.Aqui esta
 a soluçao de um colega meu de faculdade:

 Seja x=a/b (com mdc(a,b)=1) o número racional em
 questão e suponha que x é diferente de 0, 1 e -1.
 Temos

 x-1/x=a/b-b/a=(a^2-b^2)/(ab)=(a+b).(a-b)/(ab). (*)

 Suponha que d é um divisor comum de a e de a+b.
 Então d divide (a+b)-a=b. Como mdc(a,b)=1, temos,
 necessariamente, d=1.
 Analogamente (gosto dessa palavra):

 mdc(a,a-b)=mdc(b,a+b)=mdc(b,a-b)=1.

 Sendo assim, em (*) não existe fator comum entre
 numerador e denominador. Para que x-1/x seja inteiro
 restam as opções

 a+b=0, a-b=0, ab=1.


 1) Se a+b=0, teremos a=-b e x=a/b=-1, o que é nao pode
 por hupótese.

 2) Se a-b=0, teremos a=b e x=a/b=1, o que também não
 pode.

 3) Finalmente, se ab=1, teremos a=b=1 ou a=b=-1 e
 ocorre x=a/b=1; nao pode de novo!

 Sendo assim, não existe tal racional.

--
Henrique

=
Instruções para entrar na lista, sair da lista e usar a lista em
http://www.mat.puc-rio.br/~nicolau/olimp/obm-l.html
=


Re: [obm-l] MAIS UM PROBLEMA INTERESSANTE

2006-01-31 Por tôpico gugu
   Algumas sugestões:
i) Prove (mais ou menos no braço) que f(x)=x^5-x^4-4x^3+4x^2+2 é um polinômio
irredutível em Z[x].
ii) Conclua que, se r^n=a, onde a é racional, para alguma raiz r de f(x)=0 então
f(x) divide o polinômio x^n-a, e logo todas as raízes de f têm o mesmo módulo.
Verifique então que f tem duas raízes reais distintas entre 1 e 2, o que nos
leva a um absurdo.
   Abraços,
  Gugu

Citando Joÿe3o Silva [EMAIL PROTECTED]:

 (OBM - 1995) Mostre que a n-ésima raiz de um número racional (sendo n um
 inteiro positivo) não pode ser raiz do polinômio x^5 - x^4 - 4x^3 + 4x^2 + 2.




 -
  Yahoo! doce lar. Faça do Yahoo! sua homepage.





This message was sent using IMP, the Internet Messaging Program.

=
Instruções para entrar na lista, sair da lista e usar a lista em
http://www.mat.puc-rio.br/~nicolau/olimp/obm-l.html
=


Re: [obm-l] MAIS UM PROBLEMA INTERESSANTE

2006-01-31 Por tôpico Klaus Ferraz
Olá mestre,   nao entendi como provo que o polinomio (x)=x^5-x^4-4x^3+4x^2+2 é um polinômio irredutível em Z[x].[EMAIL PROTECTED] escreveu:  Algumas sugestões:i) Prove (mais ou menos no braço) que f(x)=x^5-x^4-4x^3+4x^2+2 é um polinômioirredutível em Z[x].ii) Conclua que, se r^n=a, onde a é racional, para alguma raiz r de f(x)=0 entãof(x) divide o polinômio x^n-a, e logo todas as raízes de f têm o mesmo módulo.Verifique então que f tem duas raízes reais distintas entre 1 e 2, o que nosleva a um absurdo.Abraços,GuguCitando Joÿe3o Silva <[EMAIL PROTECTED]>: (OBM - 1995) Mostre que a n-ésima raiz de um número racional (sendo n um inteiro positivo) não pode ser raiz do polinômio x^5 - x^4 - 4x^3 + 4x^2 + 2.
 - Yahoo! doce lar. Faça do Yahoo! sua homepage.This message was sent using IMP, the Internet Messaging Program.=Instruções para entrar na lista, sair da lista e usar a lista emhttp://www.mat.puc-rio.br/~nicolau/olimp/obm-l.html=  
		 
Yahoo! doce lar. Faça do Yahoo! sua homepage.

[obm-l] Cubo Perfeito

2006-01-31 Por tôpico Klaus Ferraz
Ache todas as solucoes inteiras de y^2=x^2-432.
		 
Yahoo! doce lar. Faça do Yahoo! sua homepage.

[obm-l] [obm-l] 3 Questões de Calculo aplicadas ao cotidiano

2006-01-31 Por tôpico João Vitor
1) Há 100 litro de água salgada em um tanque e essa água contém 70Kg de sal 
dissolvido.
   Despeja-se água pura no tanque a uma taxa de 3L/Min e o conteúdo é 
misturado

   permanentemente, mantendo-se uniforme, e escoando na mesma taxa.
   Quantos quilogramas de sal existirão no tanque após uma hora?


2) Numa certa reação Química, a taxa de conversão de uma substância é 
proporcional

   à quantidade de substância que ainda não reagiu até aquele instante.
   Após 10 min, um terço da quantidade da susbtância original já reagiu e
   20g já reagiram após 15 min. Qual era a quantidade original da 
substância?



3) Uma grandeza exibe um decaimento exponencial se ela decrescer a uma taxa
   diretamente proporcinal ao seu valor. Mostre que tal grandeza pode ser 
escrita

por uma função exponencial

 Q(t) = Q0e-kt

   onde a contante positiva Q0 mede o valor inicial presente (t=0) e k é 
uma constante

   positiva apropriada, chamada constante de decaimento.


Vlw
João Vitor
Fortaleza - CE 



=
Instruções para entrar na lista, sair da lista e usar a lista em
http://www.mat.puc-rio.br/~nicolau/olimp/obm-l.html
=


Re: [obm-l] Cubo Perfeito

2006-01-31 Por tôpico Iuri
x^2 - y^2 = (x+y)(x-y) = 432 = 2^4 * 3^3Isso ai vai dá 5*4=20 sistemas.. Basta resolvê-los. Mas deve haver maneira pra eliminar parte dessas solucoes... daqui a pouco alguem dá uma luz..
Em 31/01/06, Klaus Ferraz [EMAIL PROTECTED] escreveu:
Ache todas as solucoes inteiras de y^2=x^2-432.
		 
Yahoo! doce lar. Faça do Yahoo! sua homepage.




[obm-l] Fraude em BINGO

2006-01-31 Por tôpico jonathas ferreira
Existe a possibilidade de fraudar um bingo (não necessariamente
garantindo um resultado, mas no mínimo favorecer um grupo de
jogadores) sem intervir no sorteio, mas apenas nas cartelas.

Sendo que:

O número de campos disponíveis nas cartelas é idêntico (mas não os mesmos)
A saída de cada de cada número no sorteio é equiprovável

=
Instruções para entrar na lista, sair da lista e usar a lista em
http://www.mat.puc-rio.br/~nicolau/olimp/obm-l.html
=


[obm-l] 3 Questões de Calculo aplicadas ao cotidiano

2006-01-31 Por tôpico João Vitor

pessoal!

Qo Entenda  Q índice Zero
e-kt entenda e elevado a -kt

vlw 



=
Instruções para entrar na lista, sair da lista e usar a lista em
http://www.mat.puc-rio.br/~nicolau/olimp/obm-l.html
=


[obm-l] nº inteiros e raiz

2006-01-31 Por tôpico gustavo



1)Considere que a raiz quadrada de 
x é a , qual o valor de x de 
modo que x + 31 seja a + 
1


[obm-l] Re: [obm-l] Re: [obm-l] funçao geradora ordinari a!!!

2006-01-31 Por tôpico Marcelo Salhab Brogliato

Olá,
pode ser que esteja, suas contas para f(0) estavam quase totalmente 
corretas. Vc obteve: 2 * 1/2 = 1 .. e não 0.


Plotei o grafico usando o Graphmatica e obtive a sequencia pedida.. posso 
ter errado algo.


Abraços,
Salhab

- Original Message - 
From: Henrique Rennó [EMAIL PROTECTED]

To: obm-l@mat.puc-rio.br
Sent: Tuesday, January 31, 2006 4:31 PM
Subject: Re: [obm-l] Re: [obm-l] funçao geradora ordinaria!!!


Olá Marcelo!!!

Acredito que a fórmula encontrada não está correta. Caso eu esteja
errado me corrija.

f(x) = [ 2 ( (x+3/2)/3 - [ (x+3/2)/3 ] ) ]

f(0):
0+3/2 = 3/2
3/2/3 = 1/2
[x] é o maior inteiro menor que x
[1/2] = 0
f(0) = [2(1/2 - 0)] = 0 -- valor incorreto

f(1):
1+3/2 = 5/2
5/2/3 = 5/6
[x] é o maior inteiro menor que x
[5/6] = 0
f(0) = [2(5/6 - 0)] = 1 -- valor correto

f(2):
2+3/2 = 7/2
7/2/3 = 7/6
[x] é o maior inteiro menor que x
[7/6] = 1
f(0) = [2(7/6 - 1)] = 0 -- valor incorreto

Acho que a fórmula abaixo pode ser usada:

f(x) = [(x+3)/3] mod 2,
onde x pertence a N e [x] é o maior inteiro menor ou igual a x

f(0) = [(0+3)/3] mod 2 = [3/3] mod 2 = 1 mod 2 = 1
f(1) = [(1+3)/3] mod 2 = [4/3] mod 2 = 1 mod 2 = 1
f(2) = [(2+3)/3] mod 2 = [5/3] mod 2 = 1 mod 2 = 1
f(3) = [(3+3)/3] mod 2 = [6/3] mod 2 = 2 mod 2 = 0
f(4) = [(4+3)/3] mod 2 = [7/3] mod 2 = 2 mod 2 = 0
f(5) = [(5+3)/3] mod 2 = [8/3] mod 2 = 2 mod 2 = 0
f(6) = [(6+3)/3] mod 2 = [9/3] mod 2 = 3 mod 2 = 1
f(7) = [(7+3)/3] mod 2 = [10/3] mod 2 = 3 mod 2 = 1
f(8) = [(8+3)/3] mod 2 = [11/3] mod 2 = 3 mod 2 = 1
f(9) = [(9+3)/3] mod 2 = [12/3] mod 2 = 4 mod 2 = 0
f(10) = [(10+3)/3] mod 2 = [13/3] mod 2 = 4 mod 2 = 0
f(11) = [(11+3)/3] mod 2 = [14/3] mod 2 = 4 mod 2 = 0
f(12) = [(12+3)/3] mod 2 = [15/3] mod 2 = 5 mod 2 = 1
.
.
.

Abraços,

On 1/31/06, Marcelo Salhab Brogliato [EMAIL PROTECTED] wrote:

Olá,
então, fiz o seguinte:
f(x) = x - [x] , onde [x] é o maior inteiro menor que x

Fiz o seguinte, fiz o grafico ir até 2, ao invés de 1..
f(x) = 2(x - [x])

Então, estiquei para que ao inves do periodo ser 1, ser 3.
Então:

f(x) = 2(x/3 - [x/3])

Assim, g(x) = [f(x)] = [2(x/3 - [x/3])], x pertence aos naturais
nos da a seguencia:
g(0) = 0
g(1) = 0
g(2) = 0
g(3) = 1
g(4) = 1
g(5) = 1
g(6) = 0
e assim segue..
agora transladamos o grafico para tras..
logo:

f(x) = [ 2 ( (x+3/2)/3 - [ (x+3/2)/3 ] ) ]
nos da a sequencia desejada.

Abraços,
Salhab

- Original Message -
From: diego andres
To: obm-l@mat.puc-rio.br
Sent: Tuesday, January 31, 2006 10:50 AM
Subject: [obm-l] funçao geradora ordinaria!!!

 gostaria que alguem achasse a funcao geradora da
sequencia(1,1,1,0,0,0,1,1,1,0,0,0...).
grato Diego Andrés



Yahoo! doce lar. Faça do Yahoo! sua homepage.





--
Henrique

=
Instruções para entrar na lista, sair da lista e usar a lista em
http://www.mat.puc-rio.br/~nicolau/olimp/obm-l.html
= 


=
Instruções para entrar na lista, sair da lista e usar a lista em
http://www.mat.puc-rio.br/~nicolau/olimp/obm-l.html
=


Re: [obm-l] integral euleriana, funcao gama

2006-01-31 Por tôpico Marcelo Salhab Brogliato

Olá,
não pressupus que a é menor que zero em nenhum instance.
Se eu integrar de a até 0, não significa que a é menor que 0..
assim... integral de a até 0 daquela funcao eh exatamente menor integral de 
0 até a daquela função, que é igual: - Gamma(a) = - (a-1)!, que édiferente 
de 0.


o que eu quis dizer com Tem certeza dessa questao é: onde vc viu essa 
questao? tem certeza que esta correta? nao eh nada um pouco diferente?


abraços,
Salhab

- Original Message - 
From: Luís [EMAIL PROTECTED]

To: obm-l@mat.puc-rio.br
Sent: Tuesday, January 31, 2006 12:57 PM
Subject: Re: [obm-l] integral euleriana, funcao gama


Logo: (1/n) . Gamma[(m+1)/n] = int(a to +inf, (x-a)^m . e^[-(x-a)^n] . 
dx ) Agora é necessário mostrar que essa integral, com limites de a 
até 0, vale 0.

Mas aih estaria pressupondo que a eh menor que zero com base em q?
O ponto é: se a for inteiro, entao, a integral de a até 0 vale: -a! 
[fatorial]
se a for inteiro entao gama(a) = a!do jeito que a integral estah nao vejo 
motivo pra ser a mesma

Tem certeza dessa questao?

nao vi ninguem que fez

a, m e n são reais?

sao
=
Instrues para entrar na lista, sair da lista e usar a lista em
http://www.mat.puc-rio.br/~nicolau/olimp/obm-l.html
= 


=
Instru��es para entrar na lista, sair da lista e usar a lista em
http://www.mat.puc-rio.br/~nicolau/olimp/obm-l.html
=


[obm-l] RE: [obm-l] nº inteiros e raiz

2006-01-31 Por tôpico Hugo Musso Gualandi


temos que a raiz de x, sqrt(x) = a e que x+31=a+1

substituindo x=a^2 na segunda equacao temos que a^2 - a +30 = 0
como essa equacao nao tem raizes reais eu sou levado a acreditar que o 
problema original era x+1=a+31. Assim sendo a equacao passa a ser
a^2 - a - 30= 0, que tem raizes +6 e -5. Levando em conta a primeira raiz, e 
devido ao fato de que sqrt(36)=6, temos que x=36. espero ter sido de ajuda



From: gustavo [EMAIL PROTECTED]
Reply-To: obm-l@mat.puc-rio.br
To: obm-l@mat.puc-rio.br
Subject: [obm-l] nº inteiros e raiz
Date: Tue, 31 Jan 2006 23:18:41 -0200

 1)Considere que a raiz quadrada de x é a , qual o valor de x de modo que 
x + 31 seja  a + 1



=
Instruções para entrar na lista, sair da lista e usar a lista em
http://www.mat.puc-rio.br/~nicolau/olimp/obm-l.html
=


RE: [obm-l] Cubo Perfeito

2006-01-31 Por tôpico Hugo Musso Gualandi


reorganizando a equacao temos que
x^2 - y^2 = 432=
(x - y)(x + y) = 2*2*2*2*3*3*3

agora o problema se transforma em encontrar, com base nos fatores primos de 
432, todos as possibilidades para (x - y) e para (x +y), calculando assim x 
e y apos um simples sistema de equacoes.
Ex.: (x-y) = 2*2*2 e (x+y) = 2*3*3*3 == x=31 e y= 23 Isso eh uma solucao 
porque 529 = 961 - 432 (23^2 = 31^2 - 432)


Eu acho que no final das contas vai dar um certo trabalo para analizar todas 
as possibilidades mas nao devem ter tantas assim no final das contas. No 
entanto deve ser possivel excluir de cara algumas possibilidades que nao 
terao resultados inteiros. Ex.: a soma de (x-y) e (x+y) eh 2x, um numero 
par. Logo,  tempos que escolher (x-y) e (x+y) de maneira que ambos sejam 
pares ou ambos impares. Como existem fatores de 2 envolvids, a ultima nao eh 
possivel. assim eliminamos as possibilidades onde (x+y) ou (x-y) sao iguais 
a

1, 3, 9 e 27.


Agora so falta descobrir se tem alguma coisa com um cubo perfeito no meio 
que eu nao achei e que facilitaria a minha vida no problema ou se eu so tou 
viajando mesmo...




From: Klaus Ferraz [EMAIL PROTECTED]
Reply-To: obm-l@mat.puc-rio.br
To: obm-l@mat.puc-rio.br
Subject: [obm-l] Cubo Perfeito
Date: Tue, 31 Jan 2006 22:57:34 + (GMT)

Ache todas as solucoes inteiras de y^2=x^2-432.


-
 Yahoo! doce lar. Faça do Yahoo! sua homepage.



=
Instruções para entrar na lista, sair da lista e usar a lista em
http://www.mat.puc-rio.br/~nicolau/olimp/obm-l.html
=


RE: [obm-l] Cubo Perfeito

2006-01-31 Por tôpico Hugo Musso Gualandi


reorganizando a equacao temos que
x^2 - y^2 = 432=
(x - y)(x + y) = 2*2*2*2*3*3*3

agora o problema se transforma em encontrar, com base nos fatores primos de 
432, todos as possibilidades para (x - y) e para (x +y), calculando assim x 
e y apos um simples sistema de equacoes.
Ex.: (x-y) = 2*2*2 e (x+y) = 2*3*3*3 == x=31 e y= 23 Isso eh uma solucao 
porque 529 = 961 - 432 (23^2 = 31^2 - 432)


Eu acho que no final das contas vai dar um certo trabalo para analizar todas 
as possibilidades mas nao devem ter tantas assim no final das contas. No 
entanto deve ser possivel excluir de cara algumas possibilidades que nao 
terao resultados inteiros. Ex.: a soma de (x-y) e (x+y) eh 2x, um numero 
par. Logo,  tempos que escolher (x-y) e (x+y) de maneira que ambos sejam 
pares ou ambos impares. Como existem fatores de 2 envolvids, a ultima nao eh 
possivel. assim eliminamos as possibilidades onde (x+y) ou (x-y) sao iguais 
a

1, 3, 9 e 27.


Agora so falta descobrir se tem alguma coisa com um cubo perfeito no meio 
que eu nao achei e que facilitaria a minha vida no problema ou se eu so tou 
viajando mesmo...




From: Klaus Ferraz [EMAIL PROTECTED]
Reply-To: obm-l@mat.puc-rio.br
To: obm-l@mat.puc-rio.br
Subject: [obm-l] Cubo Perfeito
Date: Tue, 31 Jan 2006 22:57:34 + (GMT)

Ache todas as solucoes inteiras de y^2=x^2-432.


-
 Yahoo! doce lar. Faça do Yahoo! sua homepage.



=
Instruções para entrar na lista, sair da lista e usar a lista em
http://www.mat.puc-rio.br/~nicolau/olimp/obm-l.html
=